You are on page 1of 21

AP/GP Notes

Properties of Arithmetic and Geometric Progression

1.1

Arithmetic Progression (AP)

A sequence in AP has all its consecutive terms differs by a constant (common difference d).
Essential formulae:
Tn = a + (n 1)d
n
Sn = (2a + (n 1)d)
2
n
= (T1 + Tn )
2
How to prove a sequence is in AP:

Tn Tn1 = constant

NOTE:
1. It is redundant to prove that: Tn+1 Tn = Tn Tn1 , one just need to consider: Tn Tn1 .
2. It is WRONG to consider T2 T1 is a constant or T3 T2 = T2 T1 . (or for that matter,
other fixed integers)

1.2

Geometric Progression (GP)

A sequence is said to be in GP if the ratio of all its consecutive terms is a constant (this ratio is
called the common ratio r).
Unlike an AP, it makes sense to to talk about the sum to infinity S (i.e. sum of an infinite
number of terms) under certain conditions.
S of a GP exist, when |r| < 1.
Essential formulae:
Tn = arn1
a(1 rn )
(n is the total number of terms)
Sn =
1r
a
S =
(|r| < 1)
1r

To show that a sequence is in GP:

Tn
= constant
Tn1

NOTE:
1. Analogous to proving a sequence is an AP, to prove that a sequence is in GP, one need only
Tn
to consider strictly the ratio
being a constant.
Tn1
2. In the case of r > 0, the sum of infinity S serves as an upper bound to the sum of a
GP. When applied to real-life problems, S refers to the maximum theoretical value of the
sum of a GP. So, S will always be greater than the sum of any number of terms of a GP
provided that all the terms are positive (i.e. S > Sn for all positive integer n when r > 0).
3. For terms of GP to alternates between positive and negative numbers, it must be the case
that r < 0.
4. In the case when a = 0 or r = 0, the GP consists of only the zero term (which is usually
not sensible in the context of questions). If r = 1, then all the terms in the GP will be the
same (resulting in an AP of d = 0), which usually does not make sense in a question too.
5. Raising all terms of a GP to the same power will still result in a GP. i.e. Given a GP with
first term a and common ratio r, raising every term of the GP to power m will results in
another GP with first term am and common ratio rm .

Problems Categorization of AP/GP-type Questions

AP/GP questions are best learnt by example. Take note of the various type of questions and the
approach.

2.1

Showing a sequence is in AP/GP

Example 1:
A geometric sequence {an } has first term a and common ratio r. The sequence of numbers {bn }
satisfy the relation bn = ln(an ) for n Z+ . Show that {bn } is an arithmetic sequence and
determine the value of the common difference in terms of r.
Solution:
To show that {bn } is an AP, we will proceed to show that: bn bn1 = constant.
bn bn1 = ln an ln an1
an
= ln(
)
an1
= ln r

({an } is a GP

an
= r)
an1

= constant
{bn } is an arithmetic sequence with common difference = ln r.
Example 2:
The sum of the first n terms of a series is 6

2n+1
. Prove that the series is geometric and state
3n1
2

its first term and common ratio.


Solution:
To show that the series is a GP, we will show that:

Tn
= constant
Tn1

We next, need to obtain Tn from Sn .


This is done by: Tn = Sn Sn1 :
Tn = Sn Sn1
2n+1
2n
= (6 n1 ) (6 n2 )
3
3
2n
2n+1
= n2 n1
3
3
2n
2
(Factor out common factor)
= n2 (1 )
3
3
2n
= n1
3
Now:
Tn
2n1
2n
= n1 n2
Tn1
3
3
n
3n2
2
= n1 n1
3
2
2
=
3
= constant
2
The series is a GP with the common ratio r = .
3

2.2

Calculating number of terms in a sequence and applying it to


AP/GP problems

1. Calculating the Number of Integers n in an AP:


Tn = a + (n 1)d
Tn a
n=
+1
d
n=

Tn a
(Last term First term)
+ 1 OR n =
+1
d
d

Example:
Find the sum of all integers between 894 and 2341 (inclusive) which are not divisible by 3.
By using a calculator (or recalling the divisibility test for 3), we need to determine the first
integer and the last integer within that range that are divisible by 3.
Solution:
First integer divisible by 3: 894
Last integer divisible by 3: 2340
3

Number of integers between 894 and 2341 inclusive divisible by 3 =

2340 894
+ 1 = 483
3

Realize required sum is obtain by taking:


(Sum of integers from 894 to 2341) - (Sum of integers divisible by 3 within the range)
Number of integers from 894 to 2341 inclusive = (2341 894) + 1 = 1448
n
We will find both sums by using: (First term + Last term).
2
1448
Sum of integers from 894 to 2341 inclusive =
(894 + 2341) = 2, 342, 140
2
483
Sum of integers divisible by 3 from 894 to 2341 inclusive =
(894 + 2340) = 781, 011
2
Required Sum = 2, 342, 140 781, 011 = 1, 561, 129

2. The terms in the arithmetic sequence {3n + 1 : n = 1, 2, 3, ...} are grouped into sets such
that the rth bracket contains 2r terms as shown below:
{4, 7}, {10, 13, 16, 19}, {22, 25, 28, 31, 34, 37, 40, 43}, ...
(a) Show that the total number of terms in the first N brackets is 2(2N 1).
(b) Find the sum of terms in the first N brackets.
(c) Show that the first term in the N th bracket is 3(2N ) 2 and find also the last term in
the N th bracket.
(d) Find the least value of N such that the sum of terms in the N th bracket is more than
1012 .
(DHS11 P1)
Solution:
(a) Realize the number of integers in each set follows a GP:
1st set 2 integers
2nd set 4 integers
3rd set 8 integers
..
.
nth set 2n integers
The total number of integers in the first n sets:
= 2 + 22 + 23 + ... + 2n
2(1 2n )
(Sum of GP)
=
12
= 2(2N 1)
(Shown)
(b) Let the number of terms in the first N brackets be n.
n = 2(2N 1)
Note also that: 4, 7, 10, 13, 17, ... forms an AP.

Applying the formula for sum of an AP:


2(2N 1)
(2(4) + (2(2N 1) 1)(3))
2
= (2N 1)(8 + 6(2N ) 9)

n
(2a + (n 1)d) gives:
2
(Substitute n = 2(2N 1), a = 4, d = 3)

= (2N 1)(6(2N ) 1)
(c) To find the first term and last term in the N th bracket, we need to find the position
(i.e. n) of these 2 numbers in the AP.
Position of the first number in N th set = Position of the last number in the (N 1)th set + 1
Replace N by N 1

z
}|
{
N 1
= 2(2
1) +1
= 2N 1
First number in N th set = 4 + ((2N 1) 1)(3)
{z
}
|
a+(n1)d

= 3(2 ) 2
To find the last term in the N th set, note that the position of the last term = 2(2N 1).
So:
Position of the last number in N th set = 2(2N 1)
Last number in N th set = 4 + ((2(2N 1)) 1)(3)
= 6(2N ) 5
(d) The sum of all the terms in the N th bracket can be found by using the formula:
2N
(First Term + Last term) (Since the first and last term are known)
2
2N 1 (3(2N ) 2 + 6(2N ) 5)
2N 1 (9(2N ) 7) > 1012
The best way to solve it is to use the GC to generate a table of integers (instead of
sketching Graphs - as the numbers involve are very big (101 2) - necessary zooming is
involved).
Thus, using the GC:

Figure 1: Generated Table

From the GC, least integer n = 19.

2.3

Finding least integer for Sn to be within a certain percentage of


S

Finding the least integer n in a sum of a GP such that S differs from Sn by an amount, say c,
means:
|S Sn | < c
a(1 rn )
a

|<c
|
1r
1r
arn
|
|<c
(Simplify the numerator)
1r
a
|a|
|arn |
<c
(| | =
)

|1 r|
b
|b|
|a||r|n

<c
(|ab| = |a||b| and |rn | = |r|n )
|1 r|
c|1 r|
|r|n <
(Inequality sign remains unchanged since |1 r| and |a| positive)
|a|
n >

2.4

ln( c|1r|
)
|a|
ln |r|

(For S to exist, |r| < 1 ln |r| < 0 Flips the inequality sign)

Repeated numbers represented as sum of GP

1. Singly repeated numbers:


n amount of 3s

z }| {
Repeated numbers like 33333...333 can be represented as a sum of a GP:
3333...3333 = 3 100 + 3 10 + 3 102 + ...3 10n1
3(1 10n )
=
1 10
10n 1
=
(After simplifying)
3
2. Cyclical Repeat
In the event where the numbers with recurring decimal pattern repeating over r numbers,
it can be represented as r sums of a GP as a fraction, the following example illustrates this.

Grouping all the similar numbers together forms sum to infinity of a GP with common ratio 104

7 =
3857.385

}|
{
z
3
1
2
2
(3 10 + 3 10 + ...) + (8 10 + 8 10 )
1
+ (5 10 + 5 103 + ...) + (7 100 + 7 103 + ...)
Applying the sum to infinity for each of the following series which has a common ratio of 104

z
}|
{
3 103
8 102
50
7
=
+
+
+
1 104 1 104 1 104 1 104
3000
800
50
7
= 9999 + 9999 + 9999 + 9999
10000

10000

10000

10000

30000000 8000000 500000 70000


=
+
+
+
9999{z
9999
9999}
| 9999

Cant key in directly into GC since the number is too large

38570000
9999

For the sake of completeness, there is a easier method as follows (without the concept of
GP):
7.

Let x = 0.385
10000x = 3857.3857 (This is a trick for the cancellation of decimals - we multiply by
10000 the number repeats over 4 decimal places)
Next: 10000x x = 3857 (The decimal places cancels out)
9999x = 3857
x=

3857
9999

10000x =

38570000
9999

3. Adding up singly repeated numbers


This is best illustrated with an example:
There are n 3s

z }| {
Find the sum of the first n terms of: 3 + 33 + 333 + 3333 + .... + 3333...333
Solution:
As this series is neither a GP (clearly not an AP!) nor does it belongs to the standard
series. We need to represent the series as the summation of Tr of the series.
From part (a), the rth term of the series Tr is given by: Tr =
So:

10n 1
.
3

n amount of 3s

n
X
z }| {
10r 1
3 + 33 + 333 + ... + 3333...333 =
3
r=1
=n

z }| {
n
n
X
X
1
r
1)
= (
10
3 r=1
r=1
1 10(1 10n )
= (
n)
3 | 1
{z10 }
Sum of GP

10n+1 10 n
=

27
3

Alternatively, we can convert the above sum into a GP:

n amount of 3s

z }| {
Let S = 3 + 33 + 333 + ... + 3333...333
n amount of 9s

z }| {
3S = 9 + 99 + 999 + ... + 9999...999
n amount of 0s

z }| {
3S + (1)(n) = |10 + 100 + 1000{z
+ ... + 1000...000}
GP with common ratio of 10

10
10(10 1)
= (10n 1)
10 1
9
10 n
n
S = (10 1)
27
3

3S + n =

NOTE: The first method above is more general though the second method is more elegant!

2.5

Relating terms of AP and GP that are related

Questions linking terms (or sum) of AP and GP together are best illustrated by examples:
Example 1:
A geometric series has common ratio r, and an arithmetic series has first term a and common
difference d, where a and d are non-zero. The first three terms of the geometric series are equal
to the first, fourth and sixth terms respectively of the arithmetic series.
1. Show that 3r2 5r + 2 = 0.
2. Deduce that the geometric series is convergent and find, in terms of a, the sum to infinity.
3. The sum of the first n terms of the arithmetic series is denoted by S. Given that a > 0,
find the set of possible values of n for which S exceeds 4a.
(H2 Nov 2007 P1)

1. For simplicitys sake, we will denote the first three terms of the GP and the first,fourth and
sixth terms of the AP by G1 , G2 , G3 and A1 , A4 , A6 respectively.
We want an expression in r, so our purpose here is not to get rid of r.
Thus, it will be incorrect if we start with: (Since r is the subject below)

r=

a + 3d
a + 5d
=
a + 3d
a

Note that one can find out the value of r as follows: (Here we are deviating away from the
question to illustrate how to compute r directly)

a + 3d
a + 5d
=
a + 3d
a
2
a + 5ad = a2 + 6ad + 9d2
9d2 + ad = 0
d(9d + a) = 0
d = 0 or a = 9d

But d 6= 0 (From question), a = 9d (So, we will be able to express a in terms of d)


Substitute a = 9d into the boxed expression above gives:
r=

9d + 5d
2
=
9d + 3d
3

(Substitute into

a + 5d
a + 3d
or
will do)
a + 3d
a

But finding r directly is not our purpose here. We are to show that r, whichever the value
is, must satisfy:
3r2 5r + 2 = 0
So instead of looking at r, we look at d (i.e. the common difference of the AP):
We can relate d with the 1st , 4th and 6th term of the AP by:
A6 A4
A4 A1
=
3
2
ar a
ar2 ar
=

3
2
2ar 2a = 3ar2 3a
2r 2 = 3r2 3r
3r2 5r + 2 = 0

d=

(Since A4 = A1 + 3d and A6 = A4 + 2d)


(Since A1 = G1 , A4 = G2 and A6 = G3 )
(Divide by a on both sides since a 6= 0)
(Shown)

2. To deduce that the GP is convergent, we simply show that r is a value such that |r| < 1.
This is done by directly solving for r (r is found out earlier by another approach shown
above) from the shown equation.
3r2 5r + 2 = 0
(3r 2)(r 1) = 0
2
r = or r = 1
3

If r = 1, then all the terms of the GP will be the same the common difference of the AP
d = 0. But d 6= 0 (from question).
2
r = the GP is convergent (since | 23 | < 1)
3
So:
a
The sum to infinity S =
1 23
S = 3a
3. For: S > 4a, we will have:
n
S > 4a {2a + (n 1)d} > 4a
2
a
a
n
{2a (n 1)} > 4a (Since d = )
2
9
9
n
n1
{2
} > 4a (Cancel a off from both sides)
2
9
n2 19n + 72 < 0
We will highlight 2 methods using GC (since the resultant inequality is not easy to solve algebraically and most importantly, the question did not request for an exact answer). Thus
pointing to the direct use of the GC:
Method 1: (Using GC - Drawing Graph)

Figure 2: Solving the inequality by graphing it


From the graph above, for n2 19n + 72 < 0, 5.23 < n < 13.77 (since the graph lies below
the x-axis)
Since n is an integer, the set of values of n is given by:
{n N : 6 n 13}
Method 2: (using GC - Table Generation) (Recommended Method)
Alternatively, instead of drawing graphs, one can generate a table of integers (only when
the variable takes on integer values) using the GC:

Figure 3: Table Generated


The table generated above shows that the set of values of n are:
{n N : 6 n 13}.

10

Example 2:
The sequence of non-zero numbers a1 , a2 , a3 , ... forms a convergent geometric progression. Given
11

X
X
that a1 , a3 and a2 are three consecutive terms in an arithmetic progression and
a2r = (
a2r1 )2
r=2

r=1

1. find the common ratio of the geometric progression.


2. Show that a1 = b(220 1) where b is a real constant to be determined.
(NJC 07 P2)
Solution:
1. We are asked to find r (i.e. we can relate the AP together, making the common difference
d the subject):
So:
d = a3 a1 = a2 a3
a1 r2 a1 = a1 r a1 r2
(Since {an } is a GP, a3 = a1 r2 )
r2 1 = r r2
(a1 is non-zero)
2
2r + r 1 = 0
(2r 1)(r + 1) = 0
1
r = or r = 1 (N.A. since {an } is a convergent GP)
2
2. Clearly, we shall use the fact that:

11
X

a2r

X
a2r1 )2 to find a1 :
=(
r=1

r=2
This is a GP with common ratio r=( 21 )2 and first term=a4

Sum to infinity of GP with first term =a1 and r= 12

z }| {
11
X
a2r

z
}|
{

X
(
a2r1 )2

r=2

r=1

a4 (1 (( 12 )2 )10 )
1 ( 12 )2

a1
(
)2
1 ( 12 )2
| {z }

Apply sum to infinity of GP


4a21

1
a4 (1 ( )20 ) =
2
3
3a1 3
1 20
1
(a4 = a1 r3 with r = )

r (1 ( ) ) = a21
4
2
2
1
3 1 3
( )( ) (1 20 ) = a1
4 2
2
3 220 1
a1 = 5 ( 20 )
2
2
3 20
3
a1 = 25 (2 1) (where b = 25 )
2
2

11

Example 3:
A geometric progression and an arithmetic progression has a common first term a.
26
of the sum to infinity. The
The sum of the first three terms of the geometric progression is
27
second and third terms of the geometric progression are the nth and (n + 1)th terms of the arithmetic progression respectively. Find the sum of the first 55 terms of the arithmetic progression
in terms of a.
(MJC09 P1)
Solution:
There are 2 information given:
The sum of the first three terms of the geometric progression is
a + ar + ar2 =

26
of the sum to infinity:
27

26a
27(1 r)

Apply sum of GP for first three terms

z }| {
a(1 r3 )
1r

26a
27(1 r)
26
1 r3 =
(r 6= 1 since S exist. Also, a 6= 0)
27
1
r3 =
27
1
r=
3
=

Second term of GP is the nth term of AP: ar = a + (n 1)d = (a + nd) d


Third term of GP is the (n + 1)th of AP: ar2 = a + nd
So: ar = (a + nd) d = ar2 d
| {z }
ar2

1 1
2a
1
(Since r = )
d = ar2 ar = a( ) =
9 3
9
3
So, applying sum of AP:
d

Sum of first 55th

2.6

z}|{
55
2a
terms of AP = (2a + (55 1)( ))
2
9
55
= (2a 12a)
2
= 275a

Algebra Manipulation Questions

(Usually Promos/Prelim-Style Questions) Direct Algebraic Manipulation will be put to test to


solve certain questions. The example below illustrates this:
Example 1:
The sum of the first n terms of an arithmetic progression is denoted by Sn . Given that Sn = 50
12

and S2n = 160, find the value of S5n .


Solution:
One can write down 2 statements based on the 2 given datas:
n
(2a + (n 1)d) = 50
2
= n(2a + (2n 1)d) = 160

Sn =
S2n

(Applying sum of AP, replacing n by 2n)

Now, we will focus on the aim of finding S5n .


5n
(2a + (5n 1)d)
2
n
= 5{ (2a + (n 1 + 4n)d)}
2
n
= 5{ (2a + (n 1)d + 4nd)}
2
n
= 5{ (2a + (n 1)d)} + 10n2 d
2
= 5Sn + 10n2 d
= 5(50) + 10n2 d
= 250 + 10n2 d

S5n =

The final step is to find out what is 10n2 d. This is found by looking back at the 2 given datas.
Note that the expression for Sn and S2n are almost similar (the only difference being the presence
of n and 2n). The link between Sn and S2n is establish by considering S2n and 2Sn :
S2n 2Sn = n(2a + (2n 1)d) n(2a + (n 1)d) = 160 2(50) = 60
n2 d = 60
(Everything else cancels off)
S5n = 250 + 10(60) = 850
Example 2:
The first term of a geometric progression is 8. The sum of its first ten terms is one-eighth of
the sum of the reciprocals of these terms. Show that the sum of the first seven terms of the
original geometric progression is the same as the sum of the reciprocals of the seven terms.
(SAJC08 P1)
Solution:
Note that: Reciprocals of all terms in a GP still results in a GP.
10
10
X
1X 1
It is given that a = 8 and
ar =
, where ar are terms of the GP. Fitting the 2 infor8 r=1 ar
r=1

13

mation together, we have:


1
(1 r110 )
8(1 r10 )
1
8
=
1r
8
1 1r
8(1 r10 )
1
r10 1
1
=

9 (After simplfying)
1r
64
r1
r
1
r9 =
512
1
r=
2
7
X

7
X
1
We are required to show that:
ar =
.
a
r
r=1
r=1

(Knowing that r =

1
2

and a = 8)

7
X

8(1 ( 21 )7 )
ar =
1 ( 21 )
r=1
=

8(1 ( 12 )7 )
1
2

127
)
128
127
=
8
= 16(

7
1
X
(1 27 )
1
8
=
a
12
r=1 r

1
= (27 1)
8
127
=
8
7
X
=
ar (Shown)
r=1

Example 3:
A geometric sequence {an } has first term a and common ratio r. The sequence of numbers
{bn } satisfy the relation bn = ln(an ) for n Z+ . Given that {bn } is an AP (we have shown this
N
+1
X
in page 2), Find an expression for
bn in terms of a, aN +1 and N .
n=1

Hence, obtain an expression for a1 a2 a3 ... aN +1 in terms of a, aN +1 and N .


Solution:
Since bn is an AP, and the answer is to be expressed in a, aN +1 and N , we can use the formula:

14

Sum of an AP =

N
(first term + Last term). Thus:
2
N
+1
X

bn =

n=1

N +1
(b1 + bN +1 )
2

N +1
(ln a1 + ln aN +1 )
2
N +1
=
ln(aaN +1 )
2

Realize that: an = ebn .


So:
a1 a2 ... aN +1 = eb1 eb2 ... ebN +1
= eb1 +b2 +...+bN +1
=e
=e

PN +1
n=1

N +1
2

bn

ln(aaN +1 )

= eln(aaN +1 )
= (aaN +1 )

2.7

N +1
2

N +1
2

(Since eln x = x)

Application-type questions involving AP/GP to Real-life Scenarios

Application questions that involve AP/GP are very IMPORTANT


Example 1: (Interest Rate Question involving Recurrence Relation)
John took a bank loan of $200 000 to buy a flat. The bank charges an annual interest rate
of 3% on the outstanding loan at the end of each year. John pays $1000 at the beginning of each
month until he finishes paying for his loan. Let un denote the amount owed by John at the end
of nth year, where n Z+ .
1. Show that un = k(un1 12000), where k is a constant to be determined.
2. Express un in the form of a + (1.03n )b , where a and b are constants to be determined.
3. Find the minimum number of years required for John to pay up the bank loan.
4. Suppose John decides to terminate his loan after 15 years by paying the remaining sum by
cash. However, there is a penalty of 5% of the remaining loan for early termination. If John
had not terminated his loan earlier, find the total interest he has to pay after 15 years.
Determine, with justification, if it is to Johns benefit to make an early termination.
(HCI08 P2)
1. Note that:
Amount owned at the end of (n 1)th year = Amount owned at the beginning of nth year
Amount outstanding at the end of nth year =
Interest rate of 3%
z}|{
1.03
(amount owned at the beginning of nth year Amount paid in 1 year by the
15

end of that year)


Note that the total amount paid at the end of 1 year = $1000 12 = $12, 000
un = 1.03(un1 12000)
2. Questions of the recurrence nature is done by writing a few terms out (in this context, for
the first few years), and observing the pattern: (This is an important technique!)
Outstanding amount at:
The start of the 1st year = 200, 000
The end of the 1st year = 1.03(200, 000 12, 000) (Here, we do not evaluate the sum so as
to observe the pattern)
The end of the 2nd year
= 1.03{1.03(200, 000 12, 000) 12, 000} (By repeated use of the recurrence relation)
= (1.03)2 (200, 000) (1.03)2 (12, 000) (1.03)(12, 000)
The end of the 3rd year
= 1.03{1.03{1.03(200, 000 12, 000) 12, 000}} 12, 000}
= (1.03)3 (200, 000) (1.03)3 (12, 000) (1.03)2 (12, 000) (1.03)(12, 000)
..
.
The end of the nth year
This is a GP with common ratio r= 1 and first term (1.03)(12,000)

}| 2
{
z
= (1.03)n (200, 000) ((1.03)n (12, 000) + (1.03)n1 (12, 000) + ... + (1.03)(12, 000))
(1.03)(12, 000)(1.03n 1)
= (1.03)n (200, 000)
1.03 1
n
= (1.03) (200, 000) (412, 000)(1.03n 1)
= (1.03)n (200, 000) (412, 000)(1.03)n + 412, 000
= 412, 000 (1.03)n (200, 000 412, 000)
= 412, 000 (1.03)n (212, 000)
(Grouping and simplifying like terms)
where a = 412, 000 and b = 212, 000.
3. In order for the bank loan to be paid, Outstanding amount at the nth year 0.
(Here, the sign is instead of 0 =0 due to the fact that the amount could be paid for
anytime within the nth year)
So:
412, 000 (1.03)n (212, 000) 0
There are 3 different ways to solve the above inequality: (1 algebraic way and 2 GC ways)
Method 1: (Algebraic Way)

16

Here, the algebraic approach works as taking ln on both sides solve the problem (as the
expression is not complicated - meaning the variable n is well-isolated):
412, 000 (1.03)n (212, 000) 0
(1.03)n (212, 000) 412, 000
412, 000
1.03n
212, 000
ln( 412
)
212
(ln 1.03 > 0)
n
ln 1.03
n 22.5 (to 3 s.f.)
Least n = 23
(Since n is an integer)
Method 2: (GC Method - drawing graph)
Drawing of graphs is tricky here as the numbers involve are large Windows settings
have to be adjusted to fit the graph and necessary zooming would be needed.
So:
We plot Y = 412, 000 (1.03)X (212, 000) using the GC:

Figure 4: Zooming Out Once


Realize the graph looks like a vertical line. But it is not vertical - it just represent a steep
looking curve (which resembles a vertical line due to the window settings).
From the graph above, it takes 22.48 years to clear the loan n = 23.
Method 3: (GC Method - Generating Table)
Generating Table is only useful when the variables takes on an integer.
From the GC:

Figure 5: Table Form


From the table above, n = 23.

17

4. To decide which way is more worthwhile, we shall compare the additional amount paid for
early termination and no termination.
For early termination:
Penalty paid for early termination:
(5% penalty)(Remaining Loan after 15 years)
=(0.05)(412000 (212000)(1.03)15
=$4085.54
Without early termination:
Interest after the 15th year is calculated by taking the sum of 3% of the outstanding each
year, using the GC (since the final answer is a number):
22
22
X
X
th
Interest paid after the 15 years onwards = 0.03
ur = 0.03
412, 000 212000(1.03)r
r=16

r=16

Note: We sum up to 22 years and not 23 years because the amount is paid off completely
somewhere in the 23rd year, hence the 23rd year is not subjected to the 3% interest.
From the GC:

Figure 6: Using GC to evaluate the Interest


From the GC, interest paid without early termination = $8317.42 > $4085.54
Since the interest paid without early termination > Penalty paid for early termination
It is better for John to make an early termination.
Example 2: (Applications of AP/GP to real-life situations)
In an East Asia ancient culture, there is a war monument which is formed by placing concentric
circular slabs of granite on top of each other. The slabs of granite are of decreasing measurements.
1. The first slab of the monument has diameter 200 cm and the diameter of each subsequent
slab is three-quarters the diameter of the previous slab. Show that the total sum of the
cross-sectional circular area of the slabs used will be less than 72000cm2 , no matter how
many slabs there are.
2. The first slab of the monument has thickness 50 cm and each subsequent slab has thickness
d cm less than that of the previous slab. Given that the maximum possible number of slabs
is 14, find the largest integer value of d.

18

3. Given that d = 3, write down an expression in terms of n for the volume of the nth slab and
hence evaluate the total volume of the monument which consist of 14 slabs.
(CJC10 P1)
1. Let the sum of the cross-sectional area of the monument after stacking the nth slab be Sn .
r
200 2
)
(Since A = r2 and d = ) and common
Sn is the sum of GP with first term a = (
2
2
3 2
ratio r = ( )
4
Realize that the question is asking to show that:
Refers to Sn for any n

}|
{
z
No matter how many slabs there are < S of GP 72, 000
It suffices to show that: S 72, 000, So:

(100)2
=
=
1 ( 34 )2

Calculated using a GC

z }| {
71807.83

< 72, 000 (Shown)


nth term of AP

2. The thickness of a slab follows an AP, so the thickness of the nth

z
}|
{
slab is: 50 (n 1)d

Observe that as n increases, the thickness decreases. d affects how fast the decrease in
thickness is.
What is meant by the maximum value of d refers to the largest d such that the thickness
0 (Clearly!)
n=14
z}|{
Now, maximum possible number of slab is 14 14th slab is the slab with the smallest
positive thickness. We shall use this fact to form an inequality in d.
So:
Thickness = 50 (n 1)d 0
50 (14 1)d 0
13d 50
50
d
13
Maximum integer d = 3
3. Here, things are a little complicated. Both the diameter and thickness affects the volume
of each slab. The diameter and the thickness varies according to GP and AP respectively.
Thus the volume does not vary according to an AP or GP (it combines both AP and GP
together - which makes it neither an AP nor a GP or any standard series learnt thus far).
Thus, to tackle the summation, we have to write it as the sum of the rth term: (This is
the technique used to sum singly repeated numbers in Section 2.4)
Let Vn be the volume of the nth slab of the summation.
Volume of the monument (with 14 slabs) is given by:

19

Cross-sectional area of the rth slab


n
X

Vr =

14
X

r=1
n
X

r=1

}|

(10000(
|

Thickness of the rth slab

z
}|
{
9 r1
)
(50 3(r 1))
16
{z
}
Volume of the rth slab

9 r1
)
is not a standard series, and thus could not be evaluated in terms of n (and we
16
r=1
are not expected to give the answer in terms of n - the final answer is a number - pointing
to the usage of the GC)
r(

Thus, using a GC:

Figure 7: Summation input into GC


Volume of monument = 3,310,000 cm3 (to 3 s.f)

2.8

A final non-standard series involving GP

There may be times (though that is a rarity) where a non-routine series that is neither
an AP nor a GP (or for that matters, any of the standard series in the summation notes)
requires the student to express the answers in terms of n (i.e. Cannot use the GC). As the
following example shows:
Example:
Find an expression for

n
X

rxr in terms of x and n, where x is an arbitrary constant.

r=1

Solution:
n
X

rxr =x + 2x2 + 3x3 + ... + (n 1)xn1 + nxn

(We will write it as the sum of n GP s)

r=1

}|
{
z
=x + (x2 + x2 ) + (x3 + x3 + x3 ) + ... + (xn + xn + ... + xn )n
=xn
+xn + xn1
+xn + xn1 + xn2
..
+
.
n
n1
+x + x
+ xn2 + ... + x3 + x2
+xn + xn1 + xn2 + .... + x3 + x2 + x

20

number of xn

xn represented as a sum of GP

z }| {
xn (1 x)
1x

xn1 +xn

x2 +x3 +...+xn1 +xn

z
}|
{
z
}|
{
xn1 (1 x2 )
x2 (1 xn1 )
+
+... +
+
1x
1x

x+x2 +x3 +...+xn1 +xn

z }| {
x(1 xn )
1x

n number of xn+1

Apply sum of GP

}|
{ z
}|
{
z
(xn + xn1 + xn2 + ... + x2 + x) (xn+1 + xn+1 + ... + xn+1 )
=
1x
x(1xn )
n+1
nx
= 1x
1x
x(1 xn+1 ) nxn+1 (1 x)
=
(Combine fractions in the numerator)
(1 x)2
x (n + 1)xn+1 + nxn+2
=
(1 x)2

21

You might also like